Aufgaben:Aufgabe 2.6: Komplexe Fourierreihe: Unterschied zwischen den Versionen

Aus LNTwww
Wechseln zu:Navigation, Suche
 
(3 dazwischenliegende Versionen desselben Benutzers werden nicht angezeigt)
Zeile 5: Zeile 5:
 
[[Datei:P_ID312__Sig_A_2_6.png|right|frame|Verschiedene periodische Dreiecksignale]]
 
[[Datei:P_ID312__Sig_A_2_6.png|right|frame|Verschiedene periodische Dreiecksignale]]
  
Wir betrachten das Signal $x(t)$, das durch die beiden Parameter $T_0$ und $T_1$ festgelegt ist, wobei stets $T_1 \leq T_0$ gelten soll. Für die komplexen Fourierkoeffizienten
+
Wir betrachten das Signal  $x(t)$, das durch die beiden Parameter  $T_0$  und  $T_1$  festgelegt ist, wobei stets  $T_1 \leq T_0$  gelten soll.  Für die komplexen Fourierkoeffizienten
 
   
 
   
 
:$$D_n=\frac{1}{T_0} \cdot \int_0^{T_0}x(t)\cdot\rm e^{-\rm j\it n\omega_0t}\,{\rm d} \it t$$
 
:$$D_n=\frac{1}{T_0} \cdot \int_0^{T_0}x(t)\cdot\rm e^{-\rm j\it n\omega_0t}\,{\rm d} \it t$$
Zeile 11: Zeile 11:
 
dieses Signals erhält man nach mathematischen Umformungen:
 
dieses Signals erhält man nach mathematischen Umformungen:
 
   
 
   
:$$D_n=\frac{T_0/T_1} {(2\pi n)^2} \cdot  \bigg(1-{\rm e}^{-{\rm j} 2\pi nT_1/T_0}\bigg)-\frac{\rm j}{2\pi n}.$$
+
:$$D_n=\frac{T_0/T_1} {(2\pi n)^2} \cdot  \big(1-{\rm e}^{-{\rm j} 2\pi nT_1/T_0}\big)-\frac{\rm j}{2\pi n}.$$
  
*Der in den Teilaufgaben (1) und (3) behandelte Parametersatz (mit $T_1 = T_0/2$) ist als das Signal $x(t)$ dargestellt.  
+
*Der in den Teilaufgaben  '''(1)'''  und  '''(3)'''  behandelte Parametersatz  $($mit $T_1 = T_0/2)$  ist als das Signal  $x(t)$  dargestellt.  
*Für $T_1 = T_0$ (Teilaufgabe 2) ergibt sich die Funktion $y(t)$.  
+
*Für  $T_1 = T_0$  ⇒   Teilaufgabe  '''(2)'''  ergibt sich die Funktion  $y(t)$.  
*In der Teilaufgabe (4) wird das Signal $z(t)$ betrachtet. Dessen Fourierkoeffizienten lauten:
+
*In der Teilaufgabe  '''(4)'''  wird das Signal  $z(t)$  betrachtet. Dessen Fourierkoeffizienten lauten:
 
   
 
   
 
:$$A_0=1/4,\hspace{1cm}
 
:$$A_0=1/4,\hspace{1cm}
Zeile 25: Zeile 25:
  
  
''Hinweise:''  
+
 
*Die Aufgabe bezieht sich auf die Seite [[Signaldarstellung/Fourierreihe#Komplexe_Fourierreihe|Komplexe Fourierreihe]].
+
 
 +
''Hinweis:''  
 +
*Die Aufgabe bezieht sich auf die Seite  [[Signaldarstellung/Fourierreihe#Komplexe_Fourierreihe|Komplexe Fourierreihe]].
 
   
 
   
  
Zeile 37: Zeile 39:
 
<quiz display=simple>
 
<quiz display=simple>
  
{Berechnen Sie den Koeffizienten $D_0$ und zeigen Sie, dass dieser stets reell ist. Welcher Wert ergibt sich für $T_1 = T_0/2$, also für das Signal $x(t)$?
+
{Berechnen Sie den Koeffizienten&nbsp; $D_0$&nbsp; und zeigen Sie, dass dieser stets reell ist.&nbsp; Welcher Wert ergibt sich für&nbsp; $T_1 = T_0/2$, also für das Signal&nbsp; $x(t)$?
 
|type="{}"}
 
|type="{}"}
 
$D_0^{(x)}\ = \ $  { 0.25 3% }
 
$D_0^{(x)}\ = \ $  { 0.25 3% }
  
{Berechnen Sie für den Sonderfall $T_1 = T_0$ entsprechend dem Signal $y(t)$ die komplexen Fourierkoeffizienten $D_n^{(y)}$ für $n \neq 0$. <br>Wie lauten die Koeffizienten $A_n^{(y)}$ und $B_n^{(y)}$, insbesondere für $n = 1$?
+
{Berechnen Sie für den Sonderfall&nbsp; $T_1 = T_0$&nbsp; entsprechend dem Signal&nbsp; $y(t)$&nbsp; die komplexen Fourierkoeffizienten&nbsp; $D_n^{(y)}$&nbsp; für&nbsp; $n \neq 0$. <br>Wie lauten die Koeffizienten&nbsp; $A_n^{(y)}$&nbsp; und&nbsp; $B_n^{(y)}$,&nbsp; insbesondere für&nbsp; $n = 1$?
 
|type="{}"}
 
|type="{}"}
 
$A_1^{(y)}\ = \ $ { 0. }
 
$A_1^{(y)}\ = \ $ { 0. }
 
$B_1^{(y)}\ = \ $ { 0.318 3% }
 
$B_1^{(y)}\ = \ $ { 0.318 3% }
  
{Berechnen Sie nun für das Signal $x(t)$ mit $T_1 = T_0/2$ die Koeffizienten $A_n^{(x)}$ und $B_n^{(x)}$ für $n \neq 0$. Welche Werte ergeben sich für $A_1^{(x)}$ und $B_1^{(x)}$?
+
{Berechnen Sie nun für das Signal&nbsp; $x(t)$&nbsp; mit&nbsp; $T_1 = T_0/2$&nbsp; die Koeffizienten&nbsp; $A_n^{(x)}$&nbsp; und&nbsp; $B_n^{(x)}$&nbsp; für&nbsp; $n \neq 0$.&nbsp; Welche Werte ergeben sich für&nbsp; $A_1^{(x)}$&nbsp; und&nbsp; $B_1^{(x)}$?
 
|type="{}"}
 
|type="{}"}
 
$A_1^{(x)}\ = \ $ { 0.203 3% }
 
$A_1^{(x)}\ = \ $ { 0.203 3% }
 
$B_1^{(x)}\ = \ $ { 0.318 3% }
 
$B_1^{(x)}\ = \ $ { 0.318 3% }
  
{Welche der folgenden Aussagen treffen bezüglich $x(t)$, $y(t)$ und $z(t)$ zu?
+
{Welche der folgenden Aussagen treffen bezüglich&nbsp; $x(t)$,&nbsp; $y(t)$&nbsp; und&nbsp; $z(t)$&nbsp; zu?
 
|type="[]"}
 
|type="[]"}
- Es gilt $x(t) = y(t) + z(t)$.
+
- Es gilt&nbsp; $x(t) = y(t) + z(t)$.
+ Es gilt $x(t) = y(t) - z(t)$.
+
+ Es gilt&nbsp; $x(t) = y(t) - z(t)$.
- Die Cosinuskoeffizienten $A_n$ von $x(t)$ und $z(t)$ sind identisch.
+
- Die Cosinuskoeffizienten&nbsp; $A_n$&nbsp; von&nbsp; $x(t)$&nbsp; und&nbsp; $z(t)$&nbsp; sind identisch.
+ Die Cosinuskoeffizientenn $A_n$ von $x(t)$ und $z(t)$ sind betragsgleich.
+
+ Die Cosinuskoeffizienten&nbsp; $A_n$&nbsp; von&nbsp; $x(t)$&nbsp; und&nbsp; $z(t)$&nbsp; sind betragsgleich.
+ Die Sinuskoeffizienten $B_n$ von $y(t)$ und $z(t)$ sind identisch.
+
+ Die Sinuskoeffizienten&nbsp; $B_n$&nbsp; von&nbsp; $y(t)$&nbsp; und&nbsp; $z(t)$&nbsp; sind identisch.
  
  
Zeile 65: Zeile 67:
 
===Musterlösung===
 
===Musterlösung===
 
{{ML-Kopf}}
 
{{ML-Kopf}}
'''(1)'''&nbsp; Mit dem Eulerschen Satz ist der komplexe Fourierkoeffizient $D_n$ wie folgt darstellbar:
+
'''(1)'''&nbsp; Mit dem Eulerschen Satz ist der komplexe Fourierkoeffizient&nbsp; $D_n$&nbsp; wie folgt darstellbar:
  
 
:$${\rm Re} [D_n] =\frac{T_0/T_1}{(2\pi n)^2}\cdot(1-\cos(2\pi nT_1/T_0)),$$
 
:$${\rm Re} [D_n] =\frac{T_0/T_1}{(2\pi n)^2}\cdot(1-\cos(2\pi nT_1/T_0)),$$
Zeile 71: Zeile 73:
 
:$${\rm Im}[D_n] =\frac{T_0/T_1}{(2\pi n)^2} \cdot \sin(2\pi nT_1/T_0)-\frac{1}{2\pi n}.$$
 
:$${\rm Im}[D_n] =\frac{T_0/T_1}{(2\pi n)^2} \cdot \sin(2\pi nT_1/T_0)-\frac{1}{2\pi n}.$$
 
   
 
   
Mit der für kleine $\alpha$ -Werte gültigen Näherung $\text{sin}(\alpha ) \approx \alpha$ erhält man für den Imaginärteil:
+
*Mit der für kleine&nbsp; $\alpha$-Werte gültigen Näherung&nbsp; $\text{sin}(\alpha ) \approx \alpha$&nbsp; erhält man für den Imaginärteil:
  
 
:$${\rm Im}[D_n] =\frac{T_0/T_1}{(2\pi n)^2}\cdot(2\pi nT_1/T_0)-\frac{1}{2\pi n}=0.$$
 
:$${\rm Im}[D_n] =\frac{T_0/T_1}{(2\pi n)^2}\cdot(2\pi nT_1/T_0)-\frac{1}{2\pi n}=0.$$
 
   
 
   
Für den Realteil erhält man mit $\text{cos}(\alpha) \approx 1 – \alpha^{2}/2$:
+
*Für den Realteil erhält man mit&nbsp; $\text{cos}(\alpha) \approx 1 – \alpha^{2}/2$:
  
 
:$${\rm Re}[D_n] =\frac{T_0/T_1}{(2\pi n)^2}\frac{(2\pi nT_1/T_0)^2}{2}=\frac{T_1/T_0}{2}.$$
 
:$${\rm Re}[D_n] =\frac{T_0/T_1}{(2\pi n)^2}\frac{(2\pi nT_1/T_0)^2}{2}=\frac{T_1/T_0}{2}.$$
 
   
 
   
*Für $T_1 = T_0/2$ folgt daraus der Gleichsignalkoeffizient $D_0^{(x)} \hspace{0.1cm}\underline{= 0.25}$.  
+
*Für&nbsp; $T_1 = T_0/2$&nbsp; folgt daraus der Gleichsignalkoeffizient&nbsp; $D_0^{(x)} \hspace{0.1cm}\underline{= 0.25}$.  
*Mit $T_1 = T_0$ ergibt sich $D_0^{(y)} = 0.5$.  
+
*Mit&nbsp; $T_1 = T_0$&nbsp; ergibt sich&nbsp; $D_0^{(y)} = 0.5$.  
*Ein Vergleich mit den Signalen $x(t)$ und $y(t)$ auf der Angabenseite zeigen die Richtigkeit dieser Ergebnisse.
+
*Ein Vergleich mit den Signalen&nbsp; $x(t)$&nbsp; und&nbsp; $y(t)$&nbsp; auf der Angabenseite zeigen die Richtigkeit dieser Ergebnisse.
 +
 
  
  
'''(2)'''&nbsp; Es wird nun $n \neq 0$ vorausgesetzt. Mit $T_1 = T_0$ erhält man für den Realteil wegen $\text{cos}(2\pi n) = 1$:
+
'''(2)'''&nbsp; Es wird nun&nbsp; $n \neq 0$&nbsp; vorausgesetzt.&nbsp; Mit&nbsp; $T_1 = T_0$&nbsp; erhält man für den Realteil wegen&nbsp; $\text{cos}(2\pi n) = 1$:
  
 
:$${\rm Re}[D_n^{(y)}] =\frac{1}{(2\pi n)^2}\cdot(1-\cos(2\pi n))=0.$$
 
:$${\rm Re}[D_n^{(y)}] =\frac{1}{(2\pi n)^2}\cdot(1-\cos(2\pi n))=0.$$
 
   
 
   
Der Imagnärteil lautet:
+
*Der Imagnärteil lautet:
  
 
:$${\rm Im}[D_n^{(y)}] =\frac{1}{(2\pi n)^2}\cdot(\sin(2\pi n))-\frac{1}{2\pi n}.$$
 
:$${\rm Im}[D_n^{(y)}] =\frac{1}{(2\pi n)^2}\cdot(\sin(2\pi n))-\frac{1}{2\pi n}.$$
 
   
 
   
Wegen $\text{sin}(2\pi n) = 0$ folgt daraus &nbsp; ${\rm Im}[D_n] =-{1}/({2\pi n}).$ Somit ist
+
*Wegen&nbsp; $\text{sin}(2\pi n) = 0$&nbsp; folgt daraus &nbsp; ${\rm Im}[D_n] =-{1}/({2\pi n}).$ Somit ist
  
 
:$$D_n^{(y)}=\frac{-\rm j}{2\pi n}={1}/{2} \cdot (A_n- {\rm j} \cdot B_n).$$
 
:$$D_n^{(y)}=\frac{-\rm j}{2\pi n}={1}/{2} \cdot (A_n- {\rm j} \cdot B_n).$$
 
   
 
   
Der Koeffizientenvergleich liefert $A_n^{(y)} = 0$ und $B_n^{(y)} = 1/(\pi n)$, Insbesondere sind $A_1^{(y)} \hspace{0.1cm}\underline{= 0}$ und $B_1^{(y)}\hspace{0.1cm}\underline{ \approx 0.318}$.  
+
*Der Koeffizientenvergleich liefert&nbsp; $A_n^{(y)} = 0$&nbsp; und&nbsp; $B_n^{(y)} = 1/(\pi n)$. Insbesondere sind&nbsp; $A_1^{(y)} \hspace{0.1cm}\underline{= 0}$&nbsp; und&nbsp; $B_1^{(y)}\hspace{0.1cm}\underline{ \approx 0.318}$.
 +
 
 +
*Wie zu erwarten war, gilt stets $B_{-n}^{(y)} = -B_n^{(y)}$.
  
Wie zu erwarten war, gilt stets $B_{-n}^{(y)} = -B_n^{(y)}$.
 
  
  
'''(3)'''&nbsp; Aus der in der Teilaufgabe (1) berechneten allgemeinen Gleichung folgt mit $T_1/T_0 = 1/2$:
+
'''(3)'''&nbsp; Aus der in der Teilaufgabe&nbsp; '''(1)'''&nbsp; berechneten allgemeinen Gleichung folgt mit&nbsp; $T_1/T_0 = 1/2$:
  
 
:$$D_n^{(x)}=\frac{2}{(2\pi n)^2}(1-\cos(\pi n))+{\rm j}\cdot \left[\frac{2\sin(\pi n)}{(2\pi n)^2}-\frac{1}{(2\pi n)}\right].$$
 
:$$D_n^{(x)}=\frac{2}{(2\pi n)^2}(1-\cos(\pi n))+{\rm j}\cdot \left[\frac{2\sin(\pi n)}{(2\pi n)^2}-\frac{1}{(2\pi n)}\right].$$
 
   
 
   
Daraus erhält man die Cosinuskoeffizienten
+
*Daraus erhält man die Cosinuskoeffizienten
  
 
:$$A_n^{(x)}={2}\cdot{\rm Re}[D_n] =\left\{ \begin{array}{cl} {\frac{\displaystyle 2}{\displaystyle(\pi n)^2}} & {\rm  f\ddot{u}r\; ungeradzahliges\; \it n ,} \\ 0 &  {\rm f\ddot{u}r\; geradzahliges\;\it n.} \end{array}\right. $$
 
:$$A_n^{(x)}={2}\cdot{\rm Re}[D_n] =\left\{ \begin{array}{cl} {\frac{\displaystyle 2}{\displaystyle(\pi n)^2}} & {\rm  f\ddot{u}r\; ungeradzahliges\; \it n ,} \\ 0 &  {\rm f\ddot{u}r\; geradzahliges\;\it n.} \end{array}\right. $$
 
   
 
   
Die Sinuskoeffizienten lauten:
+
*Die Sinuskoeffizienten lauten:
  
 
:$$B_n^{(x)}=-2\cdot{\rm Im}[D_n] =\frac{1}{\pi n}.$$
 
:$$B_n^{(x)}=-2\cdot{\rm Im}[D_n] =\frac{1}{\pi n}.$$
 
   
 
   
Hierbei ist berücksichtigt, dass für alle ganzzahligen Werte von $n$ die Funktion $\text{sin}(n\pi ) = 0$ ist. Die jeweils ersten reellen Koeffizienten lauten  
+
*Hierbei ist berücksichtigt, dass für alle ganzzahligen Werte von&nbsp; $n$&nbsp; die Funktion&nbsp; $\text{sin}(n\pi ) = 0$&nbsp; ist.&nbsp; Die jeweils ersten reellen Koeffizienten lauten:
 
:$$A_1^{(x)} = 2/\pi^{2} \hspace{0.1cm}\underline{\approx 0.203},$$
 
:$$A_1^{(x)} = 2/\pi^{2} \hspace{0.1cm}\underline{\approx 0.203},$$
 
:$$B_1 = 1/\pi \hspace{0.1cm}\underline{\approx 0.318}.$$
 
:$$B_1 = 1/\pi \hspace{0.1cm}\underline{\approx 0.318}.$$
Zeile 119: Zeile 123:
  
 
'''(4)'''&nbsp; Richtig sind die <u>Lösungsvorschläge 2, 4 und 5</u>:
 
'''(4)'''&nbsp; Richtig sind die <u>Lösungsvorschläge 2, 4 und 5</u>:
*Das Signal $x(t)$ ist gleich der Differenz zwischen $y(t)$ und $z(t)$. Da $z(t)$ eine gerade und $y(t)$ eine ungerade Funktion ist, werden die Cosinuskoeffizienten $A_n$ allein durch die Koeffizienten des Signals $z(t)$ bestimmt, allerdings mit negativen Vorzeichen.  
+
*Das Signal&nbsp; $x(t)$&nbsp; ist gleich der Differenz zwischen&nbsp; $y(t)$&nbsp; und&nbsp; $z(t)$. Da&nbsp; $z(t)$&nbsp; eine gerade und&nbsp; $y(t)$&nbsp; eine ungerade Funktion ist, werden die Cosinuskoeffizienten&nbsp; $A_n$&nbsp; allein durch die Koeffizienten des Signals&nbsp; $z(t)$&nbsp; bestimmt, allerdings mit negativen Vorzeichen.  
*Die Sinuskoeffizienten $B_n$ stimmen vollständig mit denen von $y(t)$ überein.  
+
*Die Sinuskoeffizienten $B_n$ stimmen vollständig mit denen von&nbsp; $y(t)$&nbsp; überein.  
*Der Gleichsignalanteil von $x(t)$ ergibt sich aus der Differenz der beiden Gleichanteile von $y(t)$ und $z(t)$: &nbsp; $A_0 = 0.5 - 0.25 = 0.25$.
+
*Der Gleichsignalanteil von&nbsp; $x(t)$&nbsp; ergibt sich aus der Differenz der beiden Gleichanteile von&nbsp; $y(t)$&nbsp; und&nbsp; $z(t)$: &nbsp;  
 +
:$$A_0 = 0.5 - 0.25 = 0.25.$$  
 
{{ML-Fuß}}
 
{{ML-Fuß}}
  
 
__NOEDITSECTION__
 
__NOEDITSECTION__
 
[[Category:Aufgaben zu Signaldarstellung|^2. Periodische Signale^]]
 
[[Category:Aufgaben zu Signaldarstellung|^2. Periodische Signale^]]

Aktuelle Version vom 16. April 2021, 10:22 Uhr

Verschiedene periodische Dreiecksignale

Wir betrachten das Signal  $x(t)$, das durch die beiden Parameter  $T_0$  und  $T_1$  festgelegt ist, wobei stets  $T_1 \leq T_0$  gelten soll.  Für die komplexen Fourierkoeffizienten

$$D_n=\frac{1}{T_0} \cdot \int_0^{T_0}x(t)\cdot\rm e^{-\rm j\it n\omega_0t}\,{\rm d} \it t$$

dieses Signals erhält man nach mathematischen Umformungen:

$$D_n=\frac{T_0/T_1} {(2\pi n)^2} \cdot \big(1-{\rm e}^{-{\rm j} 2\pi nT_1/T_0}\big)-\frac{\rm j}{2\pi n}.$$
  • Der in den Teilaufgaben  (1)  und  (3)  behandelte Parametersatz  $($mit $T_1 = T_0/2)$  ist als das Signal  $x(t)$  dargestellt.
  • Für  $T_1 = T_0$  ⇒   Teilaufgabe  (2)  ergibt sich die Funktion  $y(t)$.
  • In der Teilaufgabe  (4)  wird das Signal  $z(t)$  betrachtet. Dessen Fourierkoeffizienten lauten:
$$A_0=1/4,\hspace{1cm} A_n=\left\{ \begin{array}{cl} {\frac{\displaystyle-2}{\displaystyle(\pi n)^2}} & {\rm f\ddot{u}r\; geradzahliges\; \it n \rm ,} \\ 0 & {\rm f\ddot{u}r\; ungeradzahliges\; \it n,} \end{array}\right. $$
$$B_n=0\; \;\; \rm{ f\ddot{u}r\; alle\; \it n.}$$




Hinweis:




Fragebogen

1

Berechnen Sie den Koeffizienten  $D_0$  und zeigen Sie, dass dieser stets reell ist.  Welcher Wert ergibt sich für  $T_1 = T_0/2$, also für das Signal  $x(t)$?

$D_0^{(x)}\ = \ $

2

Berechnen Sie für den Sonderfall  $T_1 = T_0$  entsprechend dem Signal  $y(t)$  die komplexen Fourierkoeffizienten  $D_n^{(y)}$  für  $n \neq 0$.
Wie lauten die Koeffizienten  $A_n^{(y)}$  und  $B_n^{(y)}$,  insbesondere für  $n = 1$?

$A_1^{(y)}\ = \ $

$B_1^{(y)}\ = \ $

3

Berechnen Sie nun für das Signal  $x(t)$  mit  $T_1 = T_0/2$  die Koeffizienten  $A_n^{(x)}$  und  $B_n^{(x)}$  für  $n \neq 0$.  Welche Werte ergeben sich für  $A_1^{(x)}$  und  $B_1^{(x)}$?

$A_1^{(x)}\ = \ $

$B_1^{(x)}\ = \ $

4

Welche der folgenden Aussagen treffen bezüglich  $x(t)$,  $y(t)$  und  $z(t)$  zu?

Es gilt  $x(t) = y(t) + z(t)$.
Es gilt  $x(t) = y(t) - z(t)$.
Die Cosinuskoeffizienten  $A_n$  von  $x(t)$  und  $z(t)$  sind identisch.
Die Cosinuskoeffizienten  $A_n$  von  $x(t)$  und  $z(t)$  sind betragsgleich.
Die Sinuskoeffizienten  $B_n$  von  $y(t)$  und  $z(t)$  sind identisch.


Musterlösung

(1)  Mit dem Eulerschen Satz ist der komplexe Fourierkoeffizient  $D_n$  wie folgt darstellbar:

$${\rm Re} [D_n] =\frac{T_0/T_1}{(2\pi n)^2}\cdot(1-\cos(2\pi nT_1/T_0)),$$
$${\rm Im}[D_n] =\frac{T_0/T_1}{(2\pi n)^2} \cdot \sin(2\pi nT_1/T_0)-\frac{1}{2\pi n}.$$
  • Mit der für kleine  $\alpha$-Werte gültigen Näherung  $\text{sin}(\alpha ) \approx \alpha$  erhält man für den Imaginärteil:
$${\rm Im}[D_n] =\frac{T_0/T_1}{(2\pi n)^2}\cdot(2\pi nT_1/T_0)-\frac{1}{2\pi n}=0.$$
  • Für den Realteil erhält man mit  $\text{cos}(\alpha) \approx 1 – \alpha^{2}/2$:
$${\rm Re}[D_n] =\frac{T_0/T_1}{(2\pi n)^2}\frac{(2\pi nT_1/T_0)^2}{2}=\frac{T_1/T_0}{2}.$$
  • Für  $T_1 = T_0/2$  folgt daraus der Gleichsignalkoeffizient  $D_0^{(x)} \hspace{0.1cm}\underline{= 0.25}$.
  • Mit  $T_1 = T_0$  ergibt sich  $D_0^{(y)} = 0.5$.
  • Ein Vergleich mit den Signalen  $x(t)$  und  $y(t)$  auf der Angabenseite zeigen die Richtigkeit dieser Ergebnisse.


(2)  Es wird nun  $n \neq 0$  vorausgesetzt.  Mit  $T_1 = T_0$  erhält man für den Realteil wegen  $\text{cos}(2\pi n) = 1$:

$${\rm Re}[D_n^{(y)}] =\frac{1}{(2\pi n)^2}\cdot(1-\cos(2\pi n))=0.$$
  • Der Imagnärteil lautet:
$${\rm Im}[D_n^{(y)}] =\frac{1}{(2\pi n)^2}\cdot(\sin(2\pi n))-\frac{1}{2\pi n}.$$
  • Wegen  $\text{sin}(2\pi n) = 0$  folgt daraus   ${\rm Im}[D_n] =-{1}/({2\pi n}).$ Somit ist
$$D_n^{(y)}=\frac{-\rm j}{2\pi n}={1}/{2} \cdot (A_n- {\rm j} \cdot B_n).$$
  • Der Koeffizientenvergleich liefert  $A_n^{(y)} = 0$  und  $B_n^{(y)} = 1/(\pi n)$. Insbesondere sind  $A_1^{(y)} \hspace{0.1cm}\underline{= 0}$  und  $B_1^{(y)}\hspace{0.1cm}\underline{ \approx 0.318}$.
  • Wie zu erwarten war, gilt stets $B_{-n}^{(y)} = -B_n^{(y)}$.


(3)  Aus der in der Teilaufgabe  (1)  berechneten allgemeinen Gleichung folgt mit  $T_1/T_0 = 1/2$:

$$D_n^{(x)}=\frac{2}{(2\pi n)^2}(1-\cos(\pi n))+{\rm j}\cdot \left[\frac{2\sin(\pi n)}{(2\pi n)^2}-\frac{1}{(2\pi n)}\right].$$
  • Daraus erhält man die Cosinuskoeffizienten
$$A_n^{(x)}={2}\cdot{\rm Re}[D_n] =\left\{ \begin{array}{cl} {\frac{\displaystyle 2}{\displaystyle(\pi n)^2}} & {\rm f\ddot{u}r\; ungeradzahliges\; \it n ,} \\ 0 & {\rm f\ddot{u}r\; geradzahliges\;\it n.} \end{array}\right. $$
  • Die Sinuskoeffizienten lauten:
$$B_n^{(x)}=-2\cdot{\rm Im}[D_n] =\frac{1}{\pi n}.$$
  • Hierbei ist berücksichtigt, dass für alle ganzzahligen Werte von  $n$  die Funktion  $\text{sin}(n\pi ) = 0$  ist.  Die jeweils ersten reellen Koeffizienten lauten:
$$A_1^{(x)} = 2/\pi^{2} \hspace{0.1cm}\underline{\approx 0.203},$$
$$B_1 = 1/\pi \hspace{0.1cm}\underline{\approx 0.318}.$$


(4)  Richtig sind die Lösungsvorschläge 2, 4 und 5:

  • Das Signal  $x(t)$  ist gleich der Differenz zwischen  $y(t)$  und  $z(t)$. Da  $z(t)$  eine gerade und  $y(t)$  eine ungerade Funktion ist, werden die Cosinuskoeffizienten  $A_n$  allein durch die Koeffizienten des Signals  $z(t)$  bestimmt, allerdings mit negativen Vorzeichen.
  • Die Sinuskoeffizienten $B_n$ stimmen vollständig mit denen von  $y(t)$  überein.
  • Der Gleichsignalanteil von  $x(t)$  ergibt sich aus der Differenz der beiden Gleichanteile von  $y(t)$  und  $z(t)$:  
$$A_0 = 0.5 - 0.25 = 0.25.$$